2
Ling's savings account increases by 1% every month from interest. He
deposits x dollars into his account. Select all of the expressions that
represent the amount of money in his account after the interest is paid for
the third month. 7.EE2
x(125)(125)(125)
x(10025)(10025)(10025)
(1.25)³x
3(10025)x)
x(1+0.25)(1+0.25)(1+0.25)
(10025)
x(0.0025)(0.0025)(0.0025)
(1+0.0025)(x)(1+0.0025)(1+0.0025) 3(1.25)(x)

2Ling's Savings Account Increases By 1% Every Month From Interest. Hedeposits X Dollars Into His Account.

Answers

Answer 1

After the third year, the amount in his account would be -

A{3} = $(125x/64).

What is interest?

Interest is calculated as a percent of the principal.

Given is that Ling's savings account increases by {1/4}% every month from interest. He deposits {x} dollars into his account.

After the first year, the amount in his account would be -

A{1} = x + x/4 = (5x/4)

A{1} = $(5x/4)                                                ... Eq { 1 }

After the second year, the amount in his account would be -

A{2} = (5x/4) + (1/4)% of (5x/4)

A{2} = 5x/4 + 5x/16

A{2} = (1/4)(5x + 5x/4)

A{2} = (1/4)(25x/4)

A{2} = $(25x/16)                                         ... Eq { 2 }

After the third year, the amount in his account would be -

A{3} = (25x/16) + (1/4) of (25x/16)

A{3} = (25x/16) + (25x/64)

A{3} = (1/16)(25x + 25x/4)

A{3} = (1/16)(125x/4)

A{3} = $(125x/64)                                 ... Eq { 3 }

Therefore, after the third year, the amount in his account would be -

A{3} = $(125x/64).

To solve more questions on percents, visit the link below

https://brainly.com/question/8265005

#SPJ9


Related Questions

6) Brand X sells 25 kg bags of mixed nuts
that contain 41% peanuts. To make their
product they combine Brand A mixed nuts
which contain 35% peanuts and Brand B
mixed nuts which contain 45% peanuts.
How much of each do they need to use?

Answers

25*0.41 = 0.35a + 0.45b
10.25 = 0.35a + 0.45b

a + b = 25
a = 25 - b

10.25 = 0.35a + 0.45b
10.25 = 0.35(25-b) + 0.45b
10.25 = 8.75 - 0.35b + 0.45b
1.5 = 0.1b
b = 15

a = 25 - 15
a = 10

Check

10.25 = 0.35a + 0.45b

10.25 = 0.35(10)+ 0.45(15)
10.25 = 3.5 + 6.75
10.25 = 10.25 :)

What value for x makes the equation −13(x + 11) + 12x = -26 true

Answers

Answer:-117

Step-by-step explanation:

-13x-143+12x=-26

-x=-26+143=117

x=-117

Is 1/3(36m+12) and 12m+4 equivalent expressions

Answers

The expressions 1/3(36m+12) and 12m+4  are equivalent

What are equivalent expressions?

Equivalent expressions are defined as algebraic expressions that have the same solution but differ in the mode of the arrangement of values.

These expressions are known to have variables, constants, terms, coefficients, and factors.

They are also made up of mathematical operations.

We have the expression;

1/3(36m+12)

Now, expand the bracket

36m/3 + 12/3

Divide the values

12m + 4

Hence, the expression is 12m + 4

Learn about algebraic expressions at: https://brainly.com/question/4344214

#SPJ1

Three friends go to a book fair. Allen spends $2.60. Maria spends 4 times as much as Allen. Akio spends $3.55 less than Maria.

How much does Akio spend?


PLS NEED THE HELP!!





































Answers

Answer:

$6.85

Step-by-step explanation:

$2.60 x 4 is 10.4 and that is how much Maria spends. Akio spends $3.55 less than Maria. So, 10.4 - 3.55 which is 6.85 and that is the answer.

Which of the following values is a rational number?
A. O √²
B. O √49+√7
C. O √225+ √169
D. O √2x √121
E. O √11x16

Answers

The expression whose value is a rational number will be √(4)² and √225+ √169. Then the correct options are A and C.

What is a rational number?

If the value of a numerical expression is terminating then they are the rational number then they are called a rational number and if the value of a numerical expression is non-terminating then they are called an irrational number.

Let's check all the options, then we have

A. √(4)² = 4, is a rational number.

B. √49 + √7 = 7 + √7, is an irrational number.

C. √225+ √169 = 25 + 13, is a rational number.

D. √2 x √121 = 11√2, is an irrational number.

E. √11 x 16 = 4√11, is an irrational number.

The expression whose value is a rational number will be √(4)² and √225+ √169. Then the correct options are A and C.

More about the rational number link is given below.

https://brainly.com/question/9466779

#SPJ1

{10x + 7y= - 12
9x+ 5y = -16

Answers

Answer: x=-4, y=4

Step-by-step explanation:

We can use elimination method to solve this problem.

Let's eliminate y.

In equation 2, use the coefficient of y (i.e 5) to multiply equation 1. DO the same for other equation. Thus,

5(10x+7y=-12)

7(9x+5y=-16)

=> 50x+35y=-60

     63x+35y= -112

clearly, y can be eliminated by subtraction.

Therefore,

-13x = 52  [-63+50  and 112-60)]

x = -4.

When x = -4,

9x+5y =-16

9(-4) +5y =-16

5y = 36-16

y=4.

Alexa earns 5% commission as a salesperson. She sold a gold necklace that cost $78. How much commission did Alexa earn?

Answers

Answer:

Step-by-step explanation:

=  5 /100  * 78 =  39 /10  = 3.9

$3.90

What is 76 degree F in C?

Answers

76 degrees Fahrenheit was found to be equal to approximately 24.4 degrees Celsius using the formula of conversion.

To convert Fahrenheit to Celsius, you can use the following formula:

Celsius = (Fahrenheit - 32) x 5/9

Plugging in 76 degrees Fahrenheit, we get:

Celsius = (76 - 32) x 5/9

Celsius = 44 x 5/9

Celsius = 220/9

Celsius ≈ 24.4

Therefore, 76 degrees Fahrenheit is approximately 24.4 degrees Celsius.

On the Fahrenheit scale, water freezes at 32 degrees Fahrenheit (°F) and boils at 212 °F, at standard atmospheric pressure.

On the Celsius scale, the freezing point of water is defined as 0 degrees Celsius (°C), and the boiling point of water is defined as 100 °C, at standard atmospheric pressure.

Learn more about Fahrenheit scale here brainly.com/question/31139

#SPJ4

What is the sum of the distinct prime factors of the number 560?
A. 20
B. 14
C. 18
D. 16
E. 1488

Answers

The sum of the distinct prime factors is (b) 14

How to determine  the sum of the distinct prime factors

From the question, we have the following parameters that can be used in our computation:

distinct prime factors of the number 560

The distinct prime factors of the number 560 are

Factors = 2, 5, 7.

When these numbers are added, we have

Sum = 2 + 5 + 7

Evaluate

Sum = 14

Hence , the sum is (b) 14

Read more about GCF at

https://brainly.com/question/12232865

#SPJ1

What is the distance d between the student and the mirror?

Answers

Using the fact that the two triangles must be similar, we will see that d = 7.5 ft

What is the distance between the student and the mirror?

To find the distance d we will use the fact that the two triangles must be similar triangles.

That means that the quotients between the legs must be equal.

For the triangle on the left side, the quotient is:

50ft/30ft

For the triangle on the right side, the quotient is:

d/height of the student.

We know that the height of the student is 4.5ft, so that quotient is:

d/4.5ft

Now we can write the equation:

d/4.5ft = 50ft/30ft

Solving this for d, we will get:

d = 4.5ft*(50ft/30ft) = 7.5 ft

Learn more about similar triangles at:

https://brainly.com/question/14285697

#SPJ1

Three numbers are given below. Use prime factorisation to determine the HCF and LCM 1848, 132 and 462​

Answers

Answer:

The HCF is the highest common factor of 1848, 132 and 462, which is 14.

Step-by-step explanation:

Prime factorisation is a mathematical method that breaks a number down into its prime factors, the smallest positive integers that are multiples of that number.The highest common factor of 1848, 132 and 462 is the smallest number that divides all three.The lowest common multiple is the smallest number that multiples all three numbers!

If the measure of angle f g h is 64 degrees and the measure of angle e g d is (4 x + 8) degrees, what is the value of x?.

Answers

By the help of congruent , the measure of angle f g h is 64 degrees and the measure of angle e g d is (4 x + 8) degrees and the value of x is 14

Things that have precisely the same size and shape are said to be congruent. The shapes and sizes need to hold true regardless of how we flip, spin, or rotate the forms. Two triangles that are identical in size and shape are said to be congruent triangles. Even if we flip, turn, or rotate one of two congruent triangles, the other two remain congruent.Two triangles must have the same angles and, as a result, must be congruent if their sides are the same.The equal sides and angles may not be in the same location if there is a turn or flip, but they are still there.

The are congruent So,

4x+8=64

Solve for x

X=14

Learn more about congruent

brainly.com/question/12413243

#SPJ4

Need help!
Use the Savings plan to find the amount A=? if monthly payments are $250 with APR=4% for 10 years.

Answers

[tex]{ \qquad\qquad\huge\underline{{\sf Answer}}} [/tex]

Here we go ~

The required formula is :

[tex] \qquad \dashrightarrow \sf \: A = P \times \dfrac{{\bigg(1 + \dfrac{APR}{n} \bigg) }^{ny} - 1}{\dfrac{APR}{n}}[/tex]

[tex] \textsf{A = Total accumulated savings } [/tex][tex] \textsf{P = regular deposit } [/tex][tex] \textsf{APR = annual percentage rate } [/tex][tex] \textsf{n = number of payment period per year} [/tex][tex] \textsf{y = number of years } [/tex]

Now, plug in the values ~

[tex] \qquad \dashrightarrow \sf \: A = 250 \times \dfrac{{\bigg(1 + \dfrac{0.04}{12} \bigg) }^{(12 \times 10)} - 1}{\dfrac{0.04}{12}}[/tex]

[tex] \qquad \dashrightarrow \sf \: A = 250 \times \dfrac{{\bigg(1 + 0.0033 \bigg) }^{(12 0)} - 1}{0.0033}[/tex]

[tex] \qquad \dashrightarrow \sf \: A = 250 \times \dfrac{{\bigg(1.0033 \bigg) }^{(12 0)} - 1}{0.0033}[/tex]

[tex] \qquad \dashrightarrow \sf \: A = 250 \times \dfrac{1.4908- 1}{0.0033}[/tex]

[tex] \qquad \dashrightarrow \sf \: A = 250 \times \dfrac{0.4908}{0.0033}[/tex]

[tex] \qquad \dashrightarrow \sf \: A = 250 \times 147.249[/tex]

[tex] \qquad \dashrightarrow \sf \: A = 250 \times 147.2498[/tex]

[tex] \qquad \dashrightarrow \sf \: A = 250 \times 147.2498[/tex]

[tex] \qquad \dashrightarrow \sf \: A =\$ 36812.45[/tex]

That's the required value ~

What are the zeros of the function f(x) = x² - 4x² - 5?
+√5 ti
±2, ti
±5, ti
O +√2 ti
You must check the box below prior to submitting your exam!

Answers

The zeros of the function f(x) = x² - 4x - 5 are given as follows:

x = -1 and x = 5.

How to solve the quadratic function?

The quadratic function for this problem is defined as follows:

f(x) = x² - 4x - 5


The coefficients are given as follows:

a = 1, b = -4, c = -5.

Then the discriminant is given as follows:

D = b² - 4ac

D = (-4)² - 4(1)(-5)

D = 36

Hence the zeros are given as follows:

x = (4 + sqrt(36))/2 = 5.x = (4 - sqrt(36))/2 = -1.

More can be learned about quadratic functions at https://brainly.com/question/1214333

#SPJ1

help help help help help help

Answers

Image attached below

which one of the following best describes the notion of the significance level of a hypothesis test?The probability of observing a sample statistic more extreme than the one actually obtained, assuming the null hypothesis is true
The probability of the type I error
The probability of the type II error

Answers

The Hypothesis test of probability of the type I error is True.

The Hypothesis test probability of the type II error is False.

The notion of the significance level of a hypothesis test is best described by the probability of observing a sample statistic more extreme than the one actually obtained, assuming the null hypothesis is true. This is also known as the "p-value." A significance level is often set beforehand, such as alpha (α) = 0.05, and if the p-value is less than alpha, the null hypothesis is rejected.

The probability of a type I error is related to the significance level. A type I error occurs when the null hypothesis is rejected when it is actually true. The probability of a type I error is represented by alpha (α) and is the level of significance that was set for the test.

The probability of a type II error is the probability of failing to reject a false null hypothesis. It is represented by beta (β) and depends on the sample size, the true value of the parameter being tested, and the level of significance set for the test.

To know about Hypothesis test:

https://brainly.com/question/17099835

#SPJ4

based on each set of triangles or parallel lines crest a proportion and solve it to find the missing value

pls help

Answers

The value of the x in the given triangle is 7.5 units.

What is proportion?

Proportion definition, comparative relation between things or magnitudes as to size, quantity, number.

Given is a triangle having sides, 12, 4 and 10 there is a missing side x we need to find is value,

Using the concept of parallel lines crest a proportion,

x / 10 = 12 / 16

x = 120 / 16

x = 7.5

Hence, the  value of the x in the given triangle is 7.5 units.

Learn more about proportion, click;

https://brainly.com/question/30657439

#SPJ1

Use the Exterior Angle Theorem to find the measure of each angle.


mangleC =

°


mangleD =

°


mangleDEC

Answers

The Exterior Angle Theorem states that the measure of an exterior angle of a triangle is equal to the sum of the measures of the 2 nonadjacent interior angles. That means that the measure of angle DEF is equal to the sum of measure of angle C and measure of angle D. Substituting known values gives you 2y + 9 + 6y + 10 = 115, then you can solve

8y + 19 = 115

8y = 96

y = 12

The measure of angle C is 33 degrees, the measure of angle D is 82 degrees, and because the measure of angle DEF is 115 degrees, its supplement angle (DEC) measures 65 degrees.

If you flip a coin 8 times, what is the best prediction possible for the number of times it will land on tails?

Answers

Each coin flip has a 1/2 chance of landing on heads or tails. For each two coin flips, there is a 1/4 chance that both coins will land on tails (1/2 * 1/2). If we add up the 1/4 probability from doing this test 8 times, we get that the number of times that both coins would land tails up is 2 out of the 8 times.

I NEED HELP ON THE FIRST QUESTION ASAP!!!!!!!!

Answers

Answer to question 1 is   [tex]\text{y} \le -\frac{2}{5}\text{x}+100[/tex]

=========================================================

Explanation:

The first task is to find the equation of the line through the given points in the table.

Pick any two points you want. I'll pick (0,100) and (50,80)

Determine the slope:

[tex](x_1,y_1) = (0,100) \text{ and } (x_2,y_2) = (50,80)\\\\m = \text{slope} = \frac{\text{rise}}{\text{run}} = \frac{\text{change in y}}{\text{change in x}}\\\\m = \frac{\text{y}_{2} - \text{y}_{1}}{\text{x}_{2} - \text{x}_{1}}\\\\m = \frac{80 - 100}{50 - 0}\\\\m = \frac{-20}{50}\\\\m = -\frac{2}{5}\\\\[/tex]

The slope is -2/5.

The y intercept is b = 100 because of the point (0,100)

We go from [tex]\text{y} = m\text{x}+b[/tex] to [tex]\text{y} = -\frac{2}{5}\text{x}+100[/tex]

Each point from the table is found on this line. For instance, let's check the point (140,44)

[tex]\text{y} = -\frac{2}{5}\text{x}+100\\\\44 = -\frac{2}{5}*140+100\\\\44 = -56+100\\\\44 = 44\\\\[/tex]

This confirms (140,44)

I'll let you check the other points.

------------------

Any point on the line mentioned represents a scenario where César spends all of the $50.

If he goes above the line, then he'll spend more than $50. If he goes below, he spends less than $50.

This means we want to shade the region below the boundary line [tex]\text{y} = -\frac{2}{5}\text{x}+100[/tex]

We'll replace the equal sign with a "less than or equal" sign

We arrive at [tex]\text{y} \le -\frac{2}{5}\text{x}+100[/tex] as the final answer to question 1.

Answer: Answer to question 1 is   =========================================================Explanation:The first task is to find the equation of the line through the given points in the table. Pick any two points you want. I'll pick (0,100) and (50,80) Determine the slope:The slope is -2/5.The y intercept is b = 100 because of the point (0,100)We go from  to Each point from the table is found on this line. For instance, let's check the point (140,44)This confirms (140,44)I'll let you check the other points.------------------Any point on the line mentioned represents a scenario where César spends all of the $50.If he goes above the line, then he'll spend more than $50. If he goes below, he spends less than $50.This means we want to shade the region below the boundary line We'll replace the equal sign with a "less than or equal" signWe arrive at  as the final answer to question 1.

Step-by-step explanation:

Help please the question is in the picture

Answers

Answer:

a) [tex]W(x) = 0.50 (3^x)[/tex]

W = tickets worth in $
x  = number of balloons popped

b) For 8 balloons popped, number of tickets = 6,561

Tickets worth = [tex]\$ 3,280.50[/tex]

Step-by-step explanation:

The relation between number of balloons popped(x) and the number of tickets is  
[tex]f(x) =3 ^ x\\\\f(1) = 3^1 = 3\\\\f(2) = 3^2 = 9\\\\f(3) = 3^3 = 27\\\\f(4) = 3^4 = 81[/tex]

each ticket is worth 50 cents

So for x  tickets

[tex]W(x) = 0.50 (3^x)[/tex]

Number of tickets for 8 balloons popped
f(x) = 3^8

= [tex]f(x) = 3^8 = 6,561[/tex]

Ticket worth = [tex]0.5 \times 6,561 = \$ 3,280.50[/tex]

30% of 750 using a table

Answers

Answer:
225

Step-by-step explanation:
30%×750 = 30×750/100 = 3×75 = 225

What is the equation of this graph

Answers

In this equation, the value of x is fixed at 2, which means that every point on the graph will have the same x-coordinate value of 2. This creates a vertical line that passes through the point (2, y), where y can be any real number.

Therefore, the proper equation for the graph presented is:

[tex]x=2[/tex]

help me answer this pls​

Answers

Since [tex]1 - \sin^{2}{\theta} = \cos^{2}{\theta}[/tex], the trigonometric expression [tex]\frac{\sin{\theta}}{\sqrt{1 - \sin^{2}{\theta}}} = \tan{\theta}[/tex] is proven.

How to prove the trigonometric expression?

The trigonometric expression for this problem is defined as follows:

[tex]\frac{\sin{\theta}}{\sqrt{1 - \sin^{2}{\theta}}} = \tan{\theta}[/tex]

The identity used for the proof is given as follows:

[tex]\sin^{2}{\theta} + \cos^{2}{\theta} = 1[/tex]

Hence:

[tex]1 - \sin^{2}{\theta} = \cos^{2}{\theta}[/tex]

Replacing on the left side of the expression, we have that:

[tex]\frac{\sin{\theta}}{\sqrt{1 - \sin^{2}{\theta}}} = \frac{\sin{\theta}}{\sqrt{cos^{2}{\theta}}}[/tex]

Taking the square root:

[tex]\frac{\sin{\theta}}{\sqrt{1 - \sin^{2}{\theta}}} = \frac{\sin{\theta}}{\cos{\theta}}[/tex]

The tangent is given as the division of the sine by the cosine, hence:

[tex]\frac{\sin{\theta}}{\cos{\theta}} = \tan{\theta}[/tex]

More can be learned about trigonometric identities at https://brainly.com/question/7331447

#SPJ1

What is terminal value formula ?

Answers

The terminal value formula helps estimate the value of a business beyond the explicit forecast period,

the terminal value formula = FCFF 6 / (WACC – Growth Rate)

We can say that in a DCF model with a five-year free cash flow projection, the terminal value formula = FCFF 6 / (WACC – Growth Rate)

It could be accountable that in finance, the terminal value (also known as “continuing value” or “horizon value” or "TV") of a security is the present value at a future point in time of all future cash flows when we expect stable growth rate forever. It is most likely often used in multi-stage discounted cash flow analysis and also allows for the limitation of cash flow projections to a several-year period; see Forecast period (finance). Forecasting results beyond such a period is impractical and exposes such projections to a variety of risks limiting their validity, primarily the great uncertainty involved in predicting industry and macroeconomic conditions beyond a few years.

To learn more about terminal value, click here:

brainly.com/question/28304505

#SPJ4

When a number is increased by 5%, the result is 25. What is the original number to the nearest tenth?

Answers

Anwser : 23.8

Step 1: Subtract 5% from 25.

25 - (5% of 25) = 25 - (0.05 x 25)

Step 2: Simplify the equation.

25 - (0.05 x 25) = 25 - 1.25

Step 3: Subtract 1.25 from 25.

25 - 1.25 = 23.75

The graph represents y = −1/4 x − 3. Which coordinate pair is NOT a solution for the equation? Responses A(−8, −1) (−8, −1) B(0, −3) (0, −3) C(−2, −4) (−2, −4) D(4, −4) (4, −4) Question 2

Answers

The equation given in the problem is shown below:

[tex]y=-\frac{1}{4} x-3[/tex]

Here are the answer choices:

a) (-8,-1)

b) (0,-3)

c) (-2,-4)

d) (4,-4)

We are asked to choose the coordinate pair that is NOT a solution, so let's just test each option!

To test answer choice "a", plug in [tex]x=-8[/tex] and [tex]y=-1[/tex] into the original equation, then simplify:

Plug in values: [tex]-1=-\frac{1}{4} (-8)-3[/tex]

Multiply: [tex]-1=2-3[/tex]

Subtract: [tex]-1=-1[/tex]

Notice that we now have a true expression, therefore option "a" IS A SOLUTION, and is thus not the correct answer.

Repeat for option "b" (0, -3):

Plug in values: [tex]-3=-\frac{1}{4}(0)-3[/tex]

Multiply: [tex]-3=-3[/tex]

Notice that we now have a true expression, therefore option "b" IS A SOLUTION, and is thus not the correct answer.

Repeat for option "c" (-2, -4):

Plug in values: [tex]-4=-\frac{1}{4}(-2)-3[/tex]

Multiply: [tex]-4=\frac{1}{2} -3[/tex]

Subtract: [tex]-4=-\frac{5}{2}[/tex]

Notice that we now have a false expression, therefore option "c" IS NOT SOLUTION, and is thus the correct answer.

To ensure we have the right option, check option "d" as well (4, -4):

Plug in values:[tex]-4=-\frac{1}{4} (4)-3[/tex]

Multiply: [tex]-4=-1-3[/tex]

Subtract: [tex]-4=-4[/tex]

Notice that we now have a false expression, therefore option "d" IS A SOLUTION, and is thus not the correct answer.

Notice that we have now checked all possible options and only option "c" was evaluated as a false expression, therefore option C is the correct choice in regard to this question.

Let me know if you have any questions!

A student has a collection of 72 baseball cards. All of the cards are stored in
an album with 8 cards on each page. Which expression can be used to find the
total number of pages of baseball cards in the student's album?

Answers

The required expression that can be used to find the total number of pages of baseball cards in the student's album is P = 72 / 8.

What are equation models?

The equation model is defined as the model of the given situation in the form of an equation using variables and constants.

To find the total number of pages of baseball cards in the student's album, we need to divide the total number of cards by the number of cards per page.

Let's call the number of pages "P". Then we have:

P = 72 / 8

So, the number of pages is equal to 9.

So, the expression that can be used to find the total number of pages of baseball cards in the student's album is P = 72 / 8.

Learn more about models here:
https://brainly.com/question/11721202
#SPJ9

A Ferris wheel has a radius of 218 feet. What is the
total distance covered if the Ferris wheel makes 6
rotations? Write an exact answer in terms of Pi.

Answers

The total distance covered by the Ferris wheel after 6 rotations is 2616π feet

What is an equation?

An equation is an expression that shows the relationship between two or more numbers and variables using mathematical operations. An equation can be linear, quadratic, cubic and so on, depending on the degree of the variable.

The Ferris wheel has a radius of 218 feet. The total distance covered if the Ferris wheel makes one rotation is the perimeter of the wheel.

Perimeter = 2π * radius = 2π * 218 feet = 436π

For 6 rotations:

Total distance covered = 6 * 436π = 2616π

The total distance is 2616π feet

Find out more on equation at: https://brainly.com/question/2972832

#SPJ1

Help !!!!!!!!!!!!!!!!!!!!!!!!!!!!!!!!!!!!!!!!
Evaluate each algebraic expression for the given values.
1. 9.8t^2 + 20t + 8 for t = -2, 0, 3.5
2. -3(2.1x - 7.9) for x = -18.1, -0.3, 14.4

Answers

The numeric values for each of the expressions are given as follows:

1. 9.8t² + 20t + 8:

t = -2: 7.2.t = 0: 8.t = 3.5: 198.05.

2. -3(2.1x - 7.9):

x = -18.1: 137.73. x = -0.3: 25.59x = 14.4: -67.02.

How to find the numeric value of a function or of an expression?

To find the numeric value of a function or of an expression, we replace each instance of the variable in the function or in the expression by the value at which we want to find the numeric value.

Then the numeric values are given as follows:

1. 9.8t² + 20t + 8:

t = -2: 9.8(-2)² + 20(-2) + 8 = 7.2.t = 0: 8. 9.8(0)² + 20(0) + 8 = 8.t = 3.5: 9.8(3.5)² + 20(3.5) + 8 = 198.05.

2. -3(2.1x - 7.9):

x = -18.1: -3 x (2.1(-18.1) - 7.9) = 137.73.x = -0.3: -3 x (2.1(-0.3) - 7.9) = 25.59.x = 14.4: -3 x (2.1(14.4) - 7.9) = -67.02.

Learn more about the numeric values of a function at brainly.com/question/28367050

#SPJ1

Other Questions
the following account balances appear in the 2024 adjusted trial balance of spiders corporation: common stock, $30,000; retained earnings, $8,000; dividends, $1,000; service revenue, $28,000; salaries expense, $16,000; and rent expense, $9,000. no common stock was issued during the year.Prepare the statement of stockholders' equity for the year ended December 31, 2021. (Amounts to be deducted should be entered with minus sign.) From the top of a 450-foot cliff, a hiker looks down at a river below. Her angle ofdepression to the near and far banks of the river are 80 and 50 respectively, asshown in the picture. Based on this information, how wide is the river? What is the message of The Ones Who Walk Away from Omelas? how did the annexation of texas contribute to the outbreak of the u.s.-mexican war? Which capability of social software allows users to use real-time information streams, status updates, and announcements?A. Content sharingB. ProfilesC. Feeds and notificationsD. Tagging and social bookmarkingE. Team workspaces [tex]1\frac{1}{3} :2\frac{1}{3}[/tex] overview, es una opcion del menu: the following code segment is intended to remove all duplicate elements in the list mylist. the procedure does not work as intended. what are two typical ways that a service can be classified? PLEASE HELP 25 POINTS!! NO LINKS information dictating the primary sequence of a polypeptide is permanently maintained within a cell in which of the following forms?O DNAO RNAO GeneO Steroids after adding ethlyene glycol to the dissolved 3-nitrophthalic acid in step 1, what is the desired temperature for the reaction? An electron moves from point i to point f, in the direction of a uniform electric field. During this motion:. What is the main cause of uremia? what is mpc environment meaning ? PLS HELP ITS DUE TODAY Imagine you are an American alive at the outbreak of WWII in 1939. Think about whether you would want the United States to get involved in the war or to not be involved in the war. Write a persuasive letter to President Roosevelt to explain why we should or should not join the war. Include terms that we learned in Unit 9 such as isolationist and pacifists. Your letter should be creative but based on historical facts that we have learned about the war in our Class Connect sessions and OMS lessons. Your letter should be at least two full paragraphs in length, 5-7 sentences each, and should follow a standard letter format with a greeting and salutation at the end. choose the statement that would best fit under an organizations integrity-based ethics code. 10. Which is the bigger portion, of a 12 inch pizza, or % of a 9 inch pizza? Give a reason for youranswer. i^{11} + i^{16} + i^{21} + i^{26} + i^{31} deloria points out the corrupt narrative surrounding wounded knee by comparing the _______________ medals of honor given out to the 7th cavalry on that day to the 3 that were awarded to the more than 64,000 south dakotans who fought in world war ii.a.12b.35c.18d.7e.26